Which one of the following, if substituted for the condition that if M is published in the fall, N must be published ...

kens on May 17, 2020

October 2015 SEC 3 Q23

Can someone explain how to approach this type of question? Thanks in advance!

Reply
Create a free account to read and take part in forum discussions.

Already have an account? log in

SamA on May 23, 2020

Hello @kenken,

These grouping games with only two groups often lend themselves to creating multiple scenarios. If you do so properly, then you can move through this game very quickly. I am focusing on the first two rules at first.

M and P cannot be published in the same season. Because there are only two seasons, this gives us only two options.

F: M
S: P
or
F: P
S: M

Now let's consider our second rule. K and N must be published in the same season. Again, this yields two options that I will build into my diagrams.

1.
F: M K N
S: P

2.
F: M
S: P K N

3.
F: P K N
S: M

4.
F: P
S: M K N

I generally won't write more than 4 diagrams, so the combination of these two rules is perfect. I will now build in the other rules.
Kf ---> Of
Mf ---> Ns (If you'll notice, this rule eliminates my first diagram. Now we are down to 3.)

2.
F: M
S: P K N

3.
F: P K N O
S: M

4.
F: P
S: M K N

For each diagram, we have most of our placement solved. We just have to worry about L and O. If you can make a setup like this, you will breeze through the game.